LSAT and Law School Admissions Forum

Get expert LSAT preparation and law school admissions advice from PowerScore Test Preparation.

User avatar
 Dana D
PowerScore Staff
  • PowerScore Staff
  • Posts: 195
  • Joined: Feb 06, 2024
|
#106321
Hey lemonade,

The stimulus tells us why we shouldn't look to unrest as a cause for the decline - the author is arguing it is an unstable climate which is the cause. The question wants us to strengthen this argument, but by choosing answer choice (B) you area actually weakening it, because you are saying that there was a lot of climate instability (the cause of the decline) but no unrest (the alleged effect of the climate instability). You are correct to equate unrest as related to the decline of the empire, but answer choice (B) says there was not unusual levels of unrest in that period.
 Luke Haqq
PowerScore Staff
  • PowerScore Staff
  • Posts: 763
  • Joined: Apr 26, 2012
|
#106367
Hi lemonade42!

Note that answer choice (B) is stating that the mentioned areas with the greatest climactic instability "did not experience unusual levels of unrest during that period."

The conclusion in this stimulus is the first sentence: "An unstable climate was probably a major cause of the fall of the Roman empire." We want something that strengthens this causal relationship, that is, something that connects climactic instability to the decline of a society. Answer choice (B) doesn't do that, because it indicates that there was a -lack- of unrest during a period of climactic instability. Answer choice (B) would look much better if the word "not" were missing.

Get the most out of your LSAT Prep Plus subscription.

Analyze and track your performance with our Testing and Analytics Package.